*NURSING > TEST BANK > Test Bank Advanced Practice Nursing in the Care of Older Adults 2nd edition Kennedy-Malone (All)

Test Bank Advanced Practice Nursing in the Care of Older Adults 2nd edition Kennedy-Malone

Document Content and Description Below

Advanced Practice Nursing in the Care of Older Adults / Edition 2 TESTBANK Chapter 1. Changes With Aging Multiple Choice Identify the choice that best completes the statement or answers the q... uestion. 1. The major impact of the physiological changes that occur with aging is: A. Reduced physiological reserve B. Reduced homeostatic mechanisms C. Impaired immunological response D. All of the above 2. The strongest evidence regarding normal physiological aging is available through: A. Randomized controlled clinical trials B. Cross-sectional studies C. Longitudinal studies D. Case control studies 3. All of the following statements are true about laboratory values in older adults except: A. Reference ranges are preferable B. Abnormal findings are often due to physiological aging C. Normal ranges may not be applicable for older adults D. Reference values are not necessarily acceptable values 4. Biochemical individuality is best described as: A. Each individual’s variation is often much greater than that of a larger group B. The unique biochemical profile of a selected population C. The truly “normal” individual—falling within average range D. Each individual’s variation is often much smaller than that of a larger group 5. Polypharmacy is best described as taking: A. More than nine medications per day B. More than five medications per day C. Even a single medication if there is not a clear indication for its use D. When a drug is given to treat the side effect of another drug 6. Pharmacokinetic changes with aging is reflective of: A. What the drug does to the body B. What the body does to the drug C. The effect at the site of action and the time and intensity of the drug D. The side effects commonly associated with the drug 7. All the following statements are false about drug absorption except: A. Antacids increase the bioavailability of digitalis B. Gastric acidity decreases with age C. Anticholinergics increase colonic motility D. Underlying chronic disease has little impact on drug absorption 8. All of the following statements are true about drug distribution in the elderly except: A. Drugs distributed in water have lower concentration B. Drugs distributed in fat have less intense, more prolonged effect C. Drugs highly protein bound have greater potential to cause an adverse drug reaction D. The fastest way to deliver a drug to the action site is by inhalation 9. Men have faster and more efficient biotransformation of drugs and this is thought to be due to: A. Less obesity rates than women B. Prostate enlargement C. Testosterone D. Less estrogen than women 10. The cytochrome p system involves enzymes that are generally: A. Inhibited by drugs B. Induced by drugs C. Inhibited or induced by drugs D. Associated with decreased liver perfusion 11. A statement not shown to be true about pharmacodynamics changes with aging is: A. Decreased sensitivity to oral anticoagulants B. Enhanced sensitivity to central nervous system drugs C. Drug responsiveness can be influenced by patient activity level D. There is a decreased sensitivity to beta blockers 12. Atypical presentation of disease in the elderly is reflected by all the following except: A. Infection without fever B. Depression without dysphoric mood C. Myocardial infarction with chest pain and diaphoresis D. Cardiac manifestations of thyroid disease 13. Functional abilities are best assessed by: A. Self-report of function B. Observed assessment of function C. A comprehensive head-to-toe examination D. Family report of function Chapter 2. Health Promotion Multiple Choice Identify the choice that best completes the statement or answers the question. 1. The leading cause of death in elderly travelers worldwide is: A. Cardiovascular disease B. Infections C. Accidents D. Malaria 2. Which of the following should be avoided in countries where food and water precautions are to be observed? A. Hot coffee B. Bottled water C. Salad buffet D. Unpeeled bananas 3. What insect precautions are not necessary to prevent insect-borne diseases in the tropics? A. Using 100% DEET on skin to prevent bites B. Treating clothes with permetherin C. Covering up exposed skin to lessen biting surface D. Taking malaria pills as directed for areas at risk for malaria 4. An example of secondary prevention you could recommend/order for older adults would be to: A. Check for fecal occult blood B. Wear seat belts in the car C. Provide foot care for a diabetic patient D. Administer a tetanus shot 5. Ali is a 72-year-old man who recently came to the U.S. from Nigeria. He reports having BCG (bacille Calmette-Guerin) vaccination as a child. Which of the following is correct regarding a tuberculin skin test? A. It should not be done at all. B. It should be read as smaller than it really is. C. Vaccination history is irrelevant; read as usual. D. It should be read as larger than it really is. 6. A 72-year-old woman and her husband are on a cross-country driving vacation. After a long day of driving, they stop for dinner. Midway through the meal, the woman becomes very short of breath, with chest pain and a feeling of panic. Which of the following problems is most likely? A. Pulmonary edema B. Heart failure C. Pulmonary embolism D. Pneumonia 7. Ivan W. is a 65-year-old man who is new to your practice. He has a history of COPD, CAD, hypertension, and type 2 diabetes mellitus. He has had no immunizations since his discharge from the military at age 25. Childhood diseases included chickenpox, measles, mumps, and “German measles.” He presents for a disease management visit. Which of the following immunizations would you recommend for Ivan? A. MMR, influenza, pneumococcal, Zostavax B. Influenza, pneumococcal, PPD, Hepatitis B C. Tdap, pneumococcal, influenza, Zostavax D. Hepatitis B, influenza, pneumococcal, Hepatitis A 8. Leo L. is a 62-year-old African American male who comes for an initial visit to your practice. Personal health history includes smoking 1 pack/day since age 11, consuming a case of beer (24 bottles) every weekend, and working as an assembler (sedentary job) for the past 10 years. Family history in first-degree relatives includes hypertension, high cholesterol, heart attack, and type 2 diabetes mellitus. Leo’s BMI is 32; BP today is 130/86. You order a fasting glucose, lipid profile, and return visit for BP check. This is an example of: A. Primary prevention B. Secondary prevention C. Tertiary prevention D. Health profiling 9. A local chapter of a nurse practitioner organization has begun planning a community-based screening for hypertension at a local congregate living facility. This population was selected on the basis of: A. A predicted decreased incidence of high blood pressure in this population B. A recognized element of high risk within this group C. Readily available treatment measures D. Achieving an administrative goal for the congregate living facility 10. Performing range of motion exercises on a client who has had a stroke is an example of which level of prevention? A. Primary prevention B. Tertiary prevention C. Secondary prevention D. Rehabilitation prevention 11. The nurse practitioner demonstrates an understanding of primary prevention of falling among the elderly through which management plan? A. Evaluate a need for assistive devices for ambulation after the client has been injured from a fall. B. Provide resources to correct hazards contributing to falls in the home environment. C. Reinforce the need to use prescribed eyeglasses to prevent further injury from falls. D. Provide information about medications, side effects, and interactions. 12. An example of an active strategy of health promotion for an individual to accomplish would be: A. Maintaining clean water in the local environment B. Introducing fluoride into the water C. Beginning a stress management program D. Maintaining a sanitary sewage system 13. You are working with an older male adult with a long history of alcohol abuse and a 30-year history of smoking. In recommending an intervention for this client, your responsibility is to: A. Make the individual abandon his own health practices and follow your recommendations B. 5 tuberculin units intradermal PPD injection and if negative repeat with same dose one week later C. Chest x-ray at the same time of PPD testing D. 5 tuberculin units intradermal PPD injection and if positive repeat same dose in one week Chapter 3. Exercise in Older Adults Multiple Choice Identify the choice that best completes the statement or answers the question. 1. Exercise recommended for older adults should include activities that: A. Conserve energy B. Restrict flexibility C. Strengthen muscles D. Are anaerobic in nature 2. Preferred amount of exercise for older adults is: A. 10 minutes of physical activity each morning B. 30 minutes per day of aerobic activity five times a week C. Any increase in physical activity over a sedentary lifestyle D. 60 minutes per day that includes 30 minutes of aerobic activity and 30 minutes of weight training five times a week 3. Which of the following medical conditions is not considered restrictive for engaging in physical activity? A. Unstable angina B. Dehydration C. Depression D. Uncontrolled tachycardia 4. The best recommendation for a patient who states they have no equipment to exercise would be: A. Sign a contract for a year’s membership to a local gym B. Borrow free weights from grandchildren C. Have a personal trainer come to the home three times a week D. Improvise with recommended objects at home that can be used 5. When the nurse practitioner recommends exercise for a sedentary older adult, which of the following pieces of advice should be considered for all types of exercise? A. Only use equipment recommended by physical trainers B. Start low and go slow C. Only group exercise is beneficial to someone who has not been active in a long time D. Focus only on one type of exercise for the first few months Chapter 4. Comprehensive Geriatric Assessment Multiple Choice Identify the choice that best completes the statement or answers the question. 1. The evidence reflects that comprehensive geriatric assessment should be conducted: A. On all individuals 65 and older B. On all individuals 75 and older C. By an inter-disciplinary team of professionals D. Targeting the vulnerable and frail elderly 2. Evidence-based geriatric assessment instruments available to the clinician: A. Are largely screening instruments to detect a condition B. Largely do not exist and if they do they have limited clinical utility C. Include screening, evaluation, and measurement instruments in multiple domains D. Have applicability in the outpatient but not the inpatient setting 3. When interviewing the older adult with a suspected dementia, it is most important that: A. Mental status be evaluated first in order to determine if the patient is a reliable historian B. The examiner use short simple questions and recognize non-verbal signs of discomfort C. Postpone the mental status evaluation for the following visit and establish a rapport first D. The clinician get in contact with a family member to obtain the history 4. Which is not considered a dimension of symptomatology? A. Onset B. Physical signs C. Location D. Absence of associated symptoms 5. The best approach to taking the health history is to: A. Start with an open-ended question B. Start with the review of systems C. Focus on the chief complaint D. Complete the history before conducting the examination 6. A review of the evidence relative to screening of the elderly reveals the highest evidence rating for: A. Vision screening B. Mammography screening C. Hearing screening D. Dementia screening 7. Assessment of vital signs in the elderly reflect: A. Errors in blood pressure measurement are rare with automated recording devices B. Shortness of breath in the elderly is rare in the older, deconditioned, and immobile patient C. Older adults prefer a 0-10 pain rating scale D. Older adults could be septic with a temperature within normal limits 8. What statement is true about nutrition intake in the elderly? A. Deficiencies in protein intake are common with aging. B. Malnutrition is the most common nutritional disorder among the elderly living in the community. C. Increased caloric consumption is needed as one ages. D. The serum albumin is a good reflection of protein stores. 9. What is reflective of functional decline in older adults? A. Functional decline is synonymous with advanced age. B. Some individuals die of “old age” but have maintained an active and healthy lifestyle. C. Instrumental activities of daily living are preserved longer than activities of daily living. D. It is always possible to prevent functional deterioration. 10. The leading cause of traumatic death in the elderly is due to: A. Motor vehicle accidents B. Pedestrian injuries C. Falls D. Burns 11. Timing of the get-and-go test enhances its sensitivity. The process should take less than: A. Thirty seconds B. Sixteen seconds C. Sixty seconds D. Ten seconds 12. A validated tool for assessing cognitive function specific to dementia is: A. Mini-cog B. Confusion assessment method C. Yesavage GDS scale D. NuDesc 13. The medical outcome study short form 36 remains the gold standard of quality of life instruments. It measures: A. Mental and social domains B. Social domain C. Physical, mental, and social domains D. Physical domain Chapter 5. Symptoms and Syndromes Multiple Choice Identify the choice that best completes the statement or answers the question. 1. The term “geriatric syndrome” is best described as: A. A condition that has multiple underlying factors and involves multiple systems B. A condition that has a discreet etiology that is difficult to pinpoint C. Significant progress has been made in understanding geriatric syndromes, especially falls and delirium D. Therapeutic management of a geriatric syndrome can be accomplished once a specific diagnosis is made 2. The anal wink reflex is used to test: A. Rectal prolapse B. Sensation and pudental nerve function C. Baseline and squeeze sphincter tone D. Fissures and fistulas 3. Atypical presentation of acute coronary syndrome is: A. Most common in Hispanic females B. More common in men C. Most common in African American men D. More common in females 4. What disease can mimic and often co-exists with myocardial infarctions in elders with coronary artery disease? A. Hypertension B. Esophageal disease C. Diabetic gastroparesis D. Vascular disease 5. Thoracic aortic dissection presents typically as: A. Sharp stabbing pain in the mid thorax B. Pleuretic chest pain and dyspnea C. Severe retrosternal chest pain that radiates to the back and both arms D. Unilateral pleuretic chest pain and dyspnea 6. Medications known to contribute to constipation include all of the following except: A. Stimulant laxatives B. Anticholinergic drugs C. Broad-spectrum antibiotics D. Iron 7. Bordetella pertussis is best characterized by: A. Sub-acute cough lasting greater than two weeks B. Acute cough associated with a coryzal symptom C. Chronic cough with post-nasal drip D. Non-productive acute cough 8. The routine testing of tuberculosis should occur in all of the following vulnerable populations except: A. Nursing home residents B. Prison inmates C. Hospitalized elderly D. Immune-compromised patients 9. Which of the following statements about fluid balance in the elderly is false? A. Total body water decreases with age. B. Thirst response decreases as a person ages. C. African Americans have higher rates of dehydration than white Americans. D. Assessment of skin turgor at the sternum is a reliable indicator of dehydration in the elderly. 10. Distinguishing delirium from dementia can be problematic since they may co-exist. The primary consideration in the differential is: A. Performance on the Mini Mental Status Exam B. The Confusion Assessment is negative C. Rapid change and fluctuating course of cognitive function D. The presence of behavioral symptoms with cognitive impairment 11. Presbystasis is best described as: A. Impairment in vestibular apparatus that causes dizziness B. Age-related disequilibrium of unknown pathology characterized by a gradual onset of difficulty walking C. The loss of high frequency tones with aging that can impair sensation D. A disorder of the inner ear characterized by vertigo 12. If dizziness has a predictable pattern associated with it, the clinician should first consider: A. Hypoglycemia B. Psychogenic etiology C. Cardiovascular cause D. Neurogenic cause 13. All of the following are considered as contributors to dysphagia except: A. Anticholinergics B. Drugs that increase reflux symptoms C. Inadequate intake of fluids with medications and meals D. Smooth muscle relaxants 14. Evidence shows that the most important predictor of a fall is: A. Prior history of a fall B. Cognitive impairment C. Gait and balance disturbance D. Proximal muscle weakness 15. The most cost-effective interventions used to prevent falls are: A. Use of sitters B. Use of alarms (bed, chair, monitors) C. Tai Chi exercises D. Home modifications and vitamin D supplements 16. Chronic fatigue syndrome is best described as: A. Fatigue that is constant, lasting more than three months B. Fatigue lasting longer than six months and not relieved by rest C. Fatigue that waxes and wanes over a period of three months D. Total exhaustion with inability to get out of bed 17. Which form of headache is bilateral? A. Cluster B. Tension C. Migraine D. Acute angle closure glaucoma 18. Microscopic hematuria is defined as: A. Twenty or more RBCs on a urine sample B. Three or more RBCs on a urine sample C. Twenty or more RBCs on three or more samples of urine D. Three or more RBCs on three or more samples of urine 19. Risk factors associated with the finding of a malignancy in a patient with hemoptysis include all of the following except: A. Male sex B. Smoking history C. Over age 40 D. Childhood asthma 20. Recent weight loss is defined as: A. loss of >10 pounds over the past 3-6 months B. loss of >2 pounds a week C. 5% weight loss in three months D. 10% weight loss in one year 21. The most common cause of disability in the elderly is due to: A. Diabetes B. Arthritis C. Heart disease D. Chronic obstructive pulmonary disease 22. Lipedema is best described as: A. Bilateral accumulation of interstitial fluid B. Bilateral distribution of fat in the lower extremities C. Fluid retention caused by a compromised lymphatic system D. Lipid molecules that break down and cause fluid retention 23. Drug-induced pruritus is distinguished because it: A. Occurs soon after a new drug is taken B. Usually is a generalized rash C. May occur right after the drug is taken or months later D. Usually involves localized circumscribed lesions 24. A form of syncope that is more common in the elderly than younger adults is: A. Vasovagal B. Carotid sinus sensitivity C. Orthostatic hypotension D. Arrhythmias 25. All of the following statements about tremor are true except: A. The most common tremor is the Parkinson tremor B. Most individuals with tremor do not seek medical attention C. Psychogenic tremor is uncommon D. Tremor is more prevalent in whites than blacks 26. Overflow incontinence is usually associated with: A. Loss of urine that occurs with urgency B. Cognitive impairment C. Weak pelvic floor muscles D. Bladder outlet obstruction 27. Wandering is best described as: A. Aimless excessive ambulatory behavior B. Purposeful excessive ambulatory behavior C. Risk-seeking behavior in the cognitively impaired D. A result of boredom in those with dementia Chapter 6. Skin and Lymphatic Disorders Multiple Choice Identify the choice that best completes the statement or answers the question. 1. Which of the following dermatological conditions results from reactivation of the dormant varicella virus? A. Tinea versicolor B. Seborrheic keratosis C. Verruca D. Herpes zoster 2. An older adult male presents with pain in his right chest wall for the past 48 hours. Upon examination, the nurse practitioner notices a vesicular eruption along the dermatome and identifies this as herpes zoster. The NP informs the gentleman that: A. All symptoms should disappear within three days B. Oral medications can dramatically reduce the duration and intensity of his symptoms C. He has chickenpox and can be contagious to his grandchildren D. He has a sexually transmitted disease 3. A 70-year-old white male comes to the clinic with a slightly raised, scaly, pink, and irregular lesion on his scalp. He is a farmer and works outside all day. You suspect actinic keratosis, but cannot rule out other lesions. What recommendation would you give him? A. Ignore the lesion, as it is associated with aging. B. Instruct him to use a nonprescription hydrocortisone cream to dry up the lesion. C. Perform a biopsy or refer to a dermatologist. D. Advise him to use a dandruff shampoo and return in one month if the lesion has not gone away. 4. The immunofluorescent antibody (IFA) is a laboratory test used to diagnose which of the following disorders? A. Tinea versicolor B. Herpes zoster C. Squamous cell carcinoma D. Human papilloma virus 5. A wound with drainage and foul odor should be cleansed with: A. Normal saline B. Hydrogen peroxide C. 20% acetic acid D. Betadine 6. A full thickness pressure ulcer is partially covered with eschar and the surrounding tissue is reddened. Which of the following is the most appropriate treatment for this condition? A. Apply an occlusive dressing B. Debride mechanically or chemically C. Saline moistened gauze dressings D. Vacuum-assisted wound closure 7. In examining the skin of your nursing home patient, you note a “stained glass” brownish mark on the face. Which of the following lesions best describes a stained glass brownish mark? A. Actinic keratosis B. Seborrhea keratosis C. Lentigo maligna D. Superficial spreading malignant melanoma 8. Patients who have an underlying tinea infection to the cellulitis should also be treated with which one of the following? A. An anti-fungal medication B. Topical steroids C. Oral steroids D. Zinc oxide 9. Identify the type of malignant melanoma that is associated with the Hutchinson’s sign of the cuticle of the finger. A. Lentigo maligna B. Acral lentiginous C. Nodular D. Superficial spreading malignant melanoma 10. A smooth round nodule with a pearly gray border and central induration best describes which skin lesion? A. Seborrheic keratosis B. Malignant melanoma C. Herpes zoster D. Basal cell carcinoma 11. Cellulitis is a deep skin infection involving the dermis and subcutaneous tissues. The nurse practitioner suspects cellulitis in a 70-year-old Asian diabetic male presenting with reddened edematous skin around his nares. Which statement below will the nurse practitioner use in her decision-making process for the differential diagnosis pertaining to reddened edematous skin? A. Cellulitis is two times more common in women B. Facial cellulitis is more common in people >55 C. There is low incidence of cellulitis in patients with diabetes D. Cellulitis is only a disease of the lower extremities of patients with known arterial insufficiency 12. An 82-year-old female has a “pimple” on his nose that occasionally bleeds and may have increased in size in the past year. The lesion is a 0.7-cm, dome-shaped, umbilicated papule with pearly translucence. There is also a hemorrhagic crust covering the central portion. Which of the following is the most likely diagnosis? A. Squamous cell carcinoma B. Basal cell carcinoma C. Keratocanthoma D. Sebaceous hyperplasia 13. Which of the following is generally not a first-line treatment for post herpetic neuralgia? A. Intrathecal methylprednisolone B. Gabapentin C. 5% lidocaine patch D. Topical capsaicin 14. A nursing home resident with a Stage 4 pressure ulcer that extends to the muscle layer and has significant undermining with heavy exudate should be treated with: A. Dry gauze dressings B. Duoderm C. Chemical debridement D. Calcium alginate dressings 15. Which of the following descriptions accurately documents cellulitis? A. Cool, erythematous, shiny hairless extremity with decreased pulse B. Scattered, erythematous ring-like lesions with clear centers C. Clearly demarcated, raised erythematous area of face D. Diffusely inflamed skin that is warm and tender to palpation 16. Asymmetrical bi-color lesion with irregular border measuring 8 mm is found on the right lower arm of an adult patient. This assessment finding is consistent with: A. Melanoma B. Basal cell carcinoma C. Leukoplakia D. Senile lentigines 17. Which of the following descriptions best illustrates assessment findings consistent with tinea capitis? A. Circular erythematous patches with papular, scaly annular borders and clear centers B. Inflamed scaly dry patches with broken hairs C. Web lesions with erythema and scaling borders D. Scaly pruritic erythematous lesions on inguinal creases 18. A hyperkeratatotic nodule formed as the result of exposure of the foot to moisture from perspiration is called: A. Hard corn B. Tinea pedis C. Soft corn D. Plantar warts (verrucae) 19. A 64-year-old male presents with an exacerbation of psoriasis. His social history includes 50-year two packs a day of cigarettes and a six-pack a week of beer. He states he had a recent sore throat, which he attributes to minding his young grandson. He reports that until recently the pruritis was only minimal. His BMI is 37. Which of the following factors most likely contributed to the acute presentation of psoriasis? A. Alcohol abuse B. Smoking C. Streptococcal infection D. Obesity 20. Treatment of complicated cellulitis of the lower extremity resulting from an anaerobe requires all of the following except: A. Extended antibiotic medication lasting at least 7-10 days B. Topical antifungal medication C. Inquiry when last tetanus toxoid booster was given D. Elevation of limb and consideration of compression bandaging Chapter 7. Head, Neck, and Face Disorders Multiple Choice Identify the choice that best completes the statement or answers the question. 1. A 64-year-old male presents with erythema of the sclera, tearing, and bilateral pruritus of the eyes. The symptoms occur intermittently throughout the year and he has associated clear nasal discharge. Which of the following is most likely because of the inflammation? A. Bacterium B. Allergen C. Virus D. Fungi 2. One of the first-line treatments in patients with allergic rhinitis is the use of nasal corticosteroid sprays. What is the anticipated onset of symptom relief with the use of these medications? A. Two weeks or more B. Immediate C. 1 to 2 days D. A minimum of one week 3. The nurse practitioner knows that antihistamines work primarily through which of the following mechanisms? A. Vasodilatation B. Blocking leukotriene effects C. Inhibiting histamine receptor sites D. Vasoconstriction 4. Cromolyn sodium may also be used in the treatment of allergic rhinitis. What is the mechanism of action of this medication? A. Mast cell stabilization B. Blocking the effects of IgE C. Leukotriene inhibition D. Histamine blockade 5. Patients that have atopic disorders are mediated by the production of IGE will have histamine stimulated as an immediate phase response. This release of histamine results in which of the following? A. Sinus pain, increased vascular permeability, and bronchodilation B. Bronchospasm, vascular permeability, and vasodilatation C. Contraction of smooth muscle, decreased vascular permeability, and vasoconstriction D. Vasodilatation, bronchodilation, and increased vascular permeability 6. Which of the following maneuvers is used to induce symptoms of benign paroxysmal positional vertigo? A. The Fukuda stepping test B. The Dix-Hallpike maneuver C. Forced hyperventilation D. The head thrust test 7. You have a patient complaining of vertigo and want to know what could be the cause. Knowing there are many causes for vertigo, you question the length of time the sensation lasts. She tells you several hours to days and is accompanied by tinnitus and hearing loss. You suspect which of the following conditions? A. Ménière’s disease B. Benign paroxysmal positional vertigo C. TIA D. Migraine 8. Sensory impairment (vision and hearing) in the elderly is associated with: A. Impaired quality of life B. Decreased function C. Increased mortality D. All of the above 9. Which of the following conditions is the leading cause of blindness in the United States? A. Macular degeneration B. Diabetic retinopathy C. Glaucoma D. Cataracts 10. A slightly elevated elastic tissue deposit in the conjunctiva that may extend to the cornea but does not cover it is known as a: A. Pterygium B. Pinguecula C. Xanthelsma D. Limbal nodule 11. A nonmodifiable risk factor for the development of cataracts is: A. Long-term exposure to ultraviolet B radiation B. High alcohol intake C. Strong family history D. Diabetes mellitus 12. Causes of sensorineural hearing loss include: A. Ototoxicity B. Ménière’s disease C. Otosclerosis D. Inner ear fistula 13. Clinical findings in patients with nonproliferation retinopathy include: A. Cotton wool spots B. Microaneursyms C. Deep hemorrhages D. Neovascularization 14. In examining the mouth of an older adult with a history of smoking, the nurse practitioner finds a suspicious oral lesion. The patient has been referred for a biopsy to be sent for pathology. Which is the most common oral precancerous lesion? A. Fictional keratosis B. Keratoacanthoma C. Lichen planus D. Leukoplakia 15. A gross screening for hearing is the whisper tests. Which cranial nerve is being tested when the nurse practitioner conducts this test? A. Cranial nerve V B. Cranial nerve VII C. Cranial nerve VIII D. Cranial nerve IV Chapter 8. Chest Disorders Multiple Choice Identify the choice that best completes the statement or answers the question. 1. In mitral stenosis, p waves may suggest: A. Left atrial enlargement B. Right atrial enlargement C. Left ventricle enlargement D. Right ventricle enlargement 2. Aortic regurgitation requires medical treatment for early signs of CHF with: A. Beta blockers B. ACE inhibitors C. Surgery D. Hospitalization 3. A key symptom of ischemic heart disease is chest pain. However, angina equivalents may include exertional dyspnea. Angina equivalents are important because: A. Women with ischemic heart disease many times do not present with chest pain B. Some patients may have no symptoms or atypical symptoms. Diagnosis may only be made at the time of an actual myocardial infarction C. Elderly patients have the most severe symptoms D. A & B only 4. The best evidence rating drugs to consider in a post myocardial infarction patient include: A. ASA, ACE/ARB, beta-blocker, aldosterone blockade B. Ace, ARB, Calcium channel blocker, ASA C. Long-acting nitrates, warfarin, ACE, and ARB D. ASA, clopidogrel, nitrates 5. A 55-year-old post-menopausal woman with a history of hypertension complains of jaw pain on heavy exertion. There were no complaints of chest pain. Her ECG indicates normal sinus rhythm without ST segment abnormalities. Your plan may include: A. Echocardiogram B. Exercise stress test C. Cardiac catheterization D. Myocardial perfusion imaging 6. Preceding a stress test, the following lab work might include: A. CBC and differential to differentiate ischemic heart disease from anemia B. Liver enzymes to rule out underlying gall bladder disease B. Stress test C. Chest x-ray D. Echocardiography 12. The aging process causes what normal physiological changes in the heart? A. The heart valve thickens and becomes rigid, secondary to fibrosis and sclerosis B. Cardiology occurs along with prolapse of the mitral valve and regurgitation C. Dilation of the right ventricle occurs with sclerosis of pulmonic and tricuspid valves D. Hypertrophy of the right ventricle 13. An older adult may present with atypical clinical signs of pneumonia. The nurse practitioner needs to be aware that the clustering of all of the following signs and symptoms may be indicative of pneumonia in an older person except: A. Bradycardia B. Malaise C. Anorexia D. Confusion 14. Which of the following statements is true concerning anti-arrhythmic drugs? A. Amiodarone is the only one not associated with increased mortality and it has a very favorable side effect profile. B. Both long-acting and short-acting calcium channel blockers are associated with an increased risk of cardiovascular morbidity and mortality. C. Most anti-arrhythmics have a low toxic/therapeutic ratio and some are exceedingly toxic. D. Anti-arrhythmic therapy should be initiated in the hospital for all patients. 15. Dan G., a 65-year-old man, presents to your primary care office for the evaluation of chest pain and left-sided shoulder pain. Pain begins after strenuous activity, including walking. Pain is characterized as dull, aching; 8/10 during activity, otherwise 0/10. Began a few months ago, intermittent, aggravated by exercise, and relieved by rest. Has occasional nausea. Pain is retrosternal, radiating to left shoulder, definitely affects quality of life by limiting activity. Pain is worse today; did not go away after he stopped walking. BP 120/80. Pulse 72 and regular. Normal heart sounds, S1 and S2, no murmurs. Which of the following differential diagnoses would be most likely? A. Musculoskeletal chest wall syndrome with radiation B. Esophageal motor disorder with radiation C. Acute cholecystitis with cholelithiasis D. Coronary artery disease with angina pectoris 16. Jose M. is a 68-year-old man who presents to your primary care practice for a physical. Jose has had type 2 diabetes mellitus for 5 years, diet controlled. His BMI is 32. Smoker, pack per day for 25 years. He denies other medical problems. Family history includes CAD, CABG x4 for father, now deceased; CHF, type 2 diabetes mellitus, HT for mother. According to the AHA/ACC guidelines, what stage is Jose? A. Stage A B. Stage B C. Stage C D. Stage D 17. Susan P., a 60-year-old woman with a 30 pack year history, presents to your primary care practice for evaluation of a persistent, daily cough with increased sputum production, worse in the morning, occurring over the past three months. She tells you, “I have the same thing, year after year.” Which of the following choices would you consider strongly in your critical thinking process? A. Seasonal allergies B. Acute bronchitis C. Bronchial asthma D. Chronic bronchitis 18. The best way to diagnose structural heart disease/dysfunction non-invasively is: A. Chest x-ray B. EKG C. Echocardiogram D. Heart catheterization 19. A common auscultatory finding in advanced CHF is: A. Systolic ejection murmur B. S3 gallop rhythm C. Friction rub D. Bradycardia 20. The organism most commonly responsible for community-acquired pneumonia in older adults is: A. Pseudomonas aeruginosa B. Staphylococcus aureus C. Proteus mirabilis D. Streptococcus pneumonia 21. A 72-year-old woman and her husband are on a cross-country driving vacation. After a long day of driving, they stop for dinner. Midway through the meal, the woman becomes very short of breath, with chest pain and a feeling of panic. Which of the following problems is most likely? A. Pulmonary edema B. Heart failure C. Pulmonary embolism D. Pneumonia Chapter 9. Peripheral Vascular Disorders Chapter 9: Peripheral Vascular Disorders Multiple Choice Identify the choice that best completes the statement or answers the question. 1. The clinician should begin the peripheral vascular exam with: A. Auscultation of the carotid arteries B. Checking for peripheral edema C. Auscultation of the heart D. Palpation of the upper extremity arteries 2. Inspection of the distal nails and nail beds should be evaluated for signs of decreased perfusion. Signs include: A. Clubbing B. Cyanosis C. Petechiae D. All of the above 3. A thorough vascular exam includes all of the following except: A. Cardiac exam B. Reproductive organ exam C. Peripheral vascular exam D. Cranial nerve exam 4. The majority of abdominal aortic aneurysms are due to: A. Trauma B. Infection C. Inflammation D. Atherosclerosis 5. The best initial screening test for abdominal aortic aneurysm is: A. Angiography B. CT scan with IV contrast C. Abdominal ultrasound D. MRI 6. The initial treatment for symptomatic abdominal aortic aneurysm involves: A. Emergency surgery B. Aggressive blood pressure control C. Watch-and-wait approach D. Cardiology consult 7. The major cause of death after an abdominal aortic aneurysm repair is: A. AAA rupture B. Hemorrhage C. Renal failure D. Myocardial infarction 8. An early symptom of peripheral artery disease is: A. Painful cramping of muscles during walking B. Pain is worse when the legs are dependent below the level of the heart C. Pain intensifies after vigorous walking is stopped D. Ulceration or gangrene occurs at the sight of minor injury 9. The hallmark of venous stasis ulcers include all of the following except: A. Lower extremity edema B. Hyperpigmentation of the lower extremities C. Ulcers on the medial or lateral malleolus D. Copious draining ulcers 10. Exercise is an essential element of peripheral arterial disease management primarily because: A. It encourages weight loss, as being overweight is a contributing factor to PAD B. Evidence shows exercise reduces smoking C. Collateral vessels are strengthened D. It improves aerobic capacity 11. Home exercise programs for peripheral arterial disease focus on: A. Walk until pain develops then stop B. Walk through the pain then stop C. Begin at a slow speed with 10 minutes a day and build up to 30 minutes D. Walk through the pain, rest until it goes away, then resume walking 12. When should surgical options for peripheral arterial disease be considered? A. Immediately after the diagnosis since the prognosis without revascularization is poor B. After two weeks of pentoxifylline (Trental) therapy C. Pain is not relieved with rest D. Pain limits the patient’s lifestyle or ulceration occurs 13. All of the following are classic signs of venous insufficiency except: A. Pain B. Dependent edema C. Hemosiderosis D. Dermatitis 14. Lower leg edema associated with pain is characteristic of: A. Venous insufficiency B. Heart failure C. Deep venous thrombosis D. Diabetes 15. The most common cause of secondary lymphedema is: A. Surgery B. Cancer treatment C. Infection D. Trauma 16. The imaging gold standard for lymphedema is: A. Lymphosonogram B. Lymphoscintigraphy C. Lymphoangiogram D. CT scan 17. Known risk factors for lymphedema include all of the following except: A. Osteoarthritis B. Rheumatoid arthritis C. Obesity D. Venous ulcer disease 18. Complete decongestive physiotherapy (CDP) works by: A. Stimulating the lymph vessels B. Breaking up subcutaneous fibrous tissue C. Redirecting lymph flow D. All of the above Chapter 10. Abdominal Disorders Multiple Choice Identify the choice that best completes the statement or answers the question. 1. Your 70-year-old patient has gastroesophageal reflux disease (GERD). After a trial of lifestyle modifications and antacids, the patient continues to have occasional mild heartburn after occasional meals and at night. The most appropriate next action is: A. Prokinetic agents B. H2 antagonists C. Proton pump inhibitors D. Surcralfate 2. An older patient reports burning pain after ingestion of many foods and large meals. What assessment would assist the nurse practitioner in making a diagnosis of GERD? A. Identification of a fluid wave B. Positive Murphy’s sign C. Palpable spleen D. Midepigastric pain that is not reproducible with palpation 3. The nurse practitioner is examining a 62-year-old female who has been complaining of lower abdominal pain. Upon auscultation, bowel sounds are high pitched and tinkling. Which of the following terms describes this finding? A. Succession splash B. Borborygmi C. Tenesmus D. Puddle sign 4. In teaching an older adult female client with end-stage renal disease her medication regimen, the nurse practitioner must include which of the following pieces of information in the treatment plan? A. Report any changes in the color of her stool B. Take iron supplement and elemental calcium with each meal C. Take iron supplement before meals and the calcium after meals D. Take calcium with a high phosphorus meal 5. A 62-year-old client presents with a complaint of fever, pain, and burning on urination. Difficulty urinating with dribbling has been increasing in the past few days. He has a feeling of pressure in his groin. On examination, his prostate is tender, boggy, and warm. A stat urinalysis reveals the presence of leukocytes and bacteria. He is allergic to sulfa drugs. His weight is 70 kg and his last serum creatinine was 1.0. While awaiting the culture and sensitivity, the nurse practitioner begins empiric treatment with which of the following? A. Trimethoprim/sulfamethroxazole DS bid x 2 weeks B. Ampicillin 250 mg PO q day for 10 days C. Nitrofurantoin 100 mg Q 12 hours for 7 days D. Ciprofloxacin 500 mg Q 12 hours for 14 days 6. A 62-year-old woman presents with a recurrent urinary tract infection. She now has a fever of 104°F and severe costovertebral tenderness with pyuria. What is the appropriate diagnosis and intervention for this patient? A. Septic arthritis and oral prednisolone is indicated B. Pyelonephritis and hospitalization is required C. Recurrent cystitis and 10 days of antibiotics are needed D. Pelvic inflammatory disease and 7 days of antibiotics are indicated 7. Which of the following drugs would be useful for the nurse practitioner to prescribe for an older adult to prevent gastric ulcers when a nonsteroidal anti-inflammatory drug is used for chronic pain management? A. Misoprostol (Cytotec) B. Cimetidine (Tagamet) C. Metronidazole (Flagyl) D. Bismuth subsalicylate (Pepto bismol) 8. A 68-year-old male reports painless rectal bleeding occasionally noted with thin pencil-like stools, but no pain with defection. He has a history of colon polyp removal 10 years ago but was lost to follow-up. The nurse practitioner’s appropriate intervention is: A. Digital rectal exam and send home with 3 hemoccult to return B. Immediate referral to gastroenterologist and colonoscopy C. Order a screening sigmoidoscopy D. Order a colonoscopy and barium enema and refer based on results 9. Asymptomatic 1+ bacteruria is found in a nursing home resident with an indwelling catheter. The nurse practitioner’s initial intervention includes: A. Assessing resident’s cognitive status and last change of the catheter/bag B. Prescribing prophylactic Bactrim 1 tablet at bedtime C. Ordering a urine culture and sensitivity and prescribing empiric treatment until results obtained D. Ordering an x-ray of the kidney, urine, and bladder 10. When counseling clients regarding the use of antidiarrheal drugs such as Imodium anti-diarrheal and Kaopectate, the nurse practitioner advises patients to: A. Use all the medication B. Do not use for possible infectious diarrhea C. Use should exceed one week for effectiveness D. These drugs provide exactly the same pharmaceutical effects 11. When teaching a group of older adults regarding prevention of gastroesophageal reflux disease symptoms, the nurse practitioner will include which of the following instructions? A. Raise the head of the bed with pillows at night and chew peppermints when symptoms of heartburn begins B. Raise the head of the bed on blocks and take the proton pump inhibitor medication at bedtime C. Sit up for an hour after taking any medication and restrict fluid intake D. Avoid food intolerances, raise head of bed on blocks, and take a proton pump inhibitor before a meal 12. A clinical clue for suspected renal artery stenosis would be: A. Decreased urine output B. Development of resistant hypertension in a previously well-controlled patient C. Retroperitoneal pain on the affected side D. Rising BUN level with normal creatinine level 13. Helicobacter pylori is implicated as a causative agent in the development of duodenal or gastric ulcers. What teaching should the nurse practitioner plan for a patient who has a positive Helicobacter pylori test? A. It is highly contagious and a mask should be worn at home. B. Treatment regimen is multiple lifetime medications. C. Treatment regimen is multiple medications taken daily for a few weeks. D. Treatment regimen is complicated and is not indicated unless the patient is symptomatic. 14. An obese middle-aged client presents with a month of nonproductive irritating cough without fever. He also reports occasional morning hoarseness. What should the differential include? A. Atypical pneumonia B. Peptic ulcer disease C. Gastroesophageal reflux D. Mononucleosis (Epstein-Barr) 15. Which of the following findings would indicate a need for another endoscopy in clients with peptic ulcer disease? A. Cases of dyspepsia with constipation B. Symptoms persisting after 6-8 weeks of therapy C. All clients with dyspepsia who smoke and drink alcohol D. When a therapeutic response to empiric treatment is obtained 16. A careful history of a female client with a chief complaint of intermittent diarrhea reveals that she also experiences bouts of constipation. She has no known allergies and experienced no unintentional weight loss. What is the most likely condition? A. Inflammatory bowel disease B. Irritable bowel syndrome C. Giardiasis D. Lactose intolerance 17. The nurse practitioner is discussing lifestyle changes with a patient diagnosed with gastroesophageal reflux. What are the nonpharmacological management interventions that should be included? A. Weight reduction and rest 30 minutes after each meal in the supine position B. Elevation of head of the bed 4-6 inches on blocks and weight reduction C. Encouraged to wear restrictive clothing to add support for diaphragmatic breathing D. Using oral mints to relieve gastric distress 18. In differentiating a gastric ulcer from a duodenal ulcer, you know that each type of ulcer can present with distinct signs and symptoms. Which of the following pieces of information from the patient’s history is the least useful for you to determine that the patient has a duodenal ulcer? A. Pain occurs on an empty stomach B. Diffuse epigastric pain C. Rarely associated with non-steroidal use D. Occurs in patients under 40 years of age 19. A 74-year-old obese female presents complaining of persistent right upper quadrant pain. She reports that she has not had any prior abdominal surgeries. Which of the following laboratory studies would be most indicative of acute cholecystitis? A. C-reactive protein level of 3 mg B. White blood cell count of 11,000 C. Direct serum bilirubin level of 0.3 mg/dL D. Serum amylase level of 145 U/L 20. Which of the following is not a contributing factor to the development of esophagitis in older adults? A. Increased gastric emptying time B. Regular ingestion of NSAIDs C. Decreased salivation D. Fungal infections such as Candida Chapter 11. Urological and Gynecologic Disorders Multiple Choice Identify the choice that best completes the statement or answers the question. 1. Which ethnic group has the highest incidence of prostate cancer? A. Asians B. Hispanics C. African Americans D. American Indians 2. Men with an initial PSA level below 2.5 ng/ml can reduce their screening frequency to what intervals? A. Every 6 months B. Yearly C. Every 2 years D. Every 2 to 4 years 3. All of the following may be reasons associated with an elevated PSA besides prostate cancer except: A. Prostatitis B. Urinary tract infection C. Perineal trauma D. Digital rectal exam 4. In the diagnosis of acute bacterial prostatitis, a midstream urine culture is of benefit. To be diagnostic, the specimen should reveal how many white blood cells per high-power field? A. Five B. Ten C. Fifteen D. Twenty 5. In chronic bacterial prostatitis, what is the organism most commonly associated with the disease? A. Klebsiella B. Proteus C. Pseudomonas D. Escherichia coli 6. In acute prostatitis, an exam of the prostate may find the gland to be: A. Nodular B. Cool and pliable C. Swollen and tender D. Asymmetrical 7. All of the following antimicrobials may be indicated in chronic bacterial prostatitis except: A. Ciprofloxacin B. Levofloxacin C. Trimethoprim D. Azithromycin 8. Age-related changes in the bladder, urethra, and ureters include all of the following in older women except: A. Increased estrogen production’s influence on the bladder and ureter B. Decline in bladder outlet function C. Decline in ureteral resistance pressure D. Laxity of the pelvic muscle 9. Mr. Jones is a 68-year-old retired Air Force pilot that has been diagnosed with prostate cancer in the past week. He has never had a surgical procedure in his life and seeks clarification on the availability of treatments for prostate cancer. He asks the nurse practitioner to tell him the side effects of a radical prostatectomy. Which of the following is NOT a potential side effect of this procedure? A. Urinary incontinence B. Impotence C. Dribbling urine D. Selected low back pain 10. The nurse practitioner is evaluating a patient’s pelvic muscle strength by digital examination. This is performed when: A. A male patient complains of nocturia and dribbling B. The nurse practitioner needs to confirm a cystocele or stress incontinence C. The patient reports sudden loss of large amounts of urine or urge incontinence D. The nurse practitioner suspects overflow incontinence 11. A pelvic mass in a postmenopausal woman: A. is most commonly due to uterine fibroids. B. is always symptomatic. C. is highly suspicious for ovarian cancer. D. should be monitored over a period of time. 12. A 79-year-old man is being evaluated for frequent urinary dribbling without burning. Physical examination reveals a smooth but slightly enlarged prostate gland. His PSA level is 3.3 ng/mL. The patient undergoes formal urodynamic studies, and findings are as follows: a decreased bladder capacity of 370 mL; a few involuntary detrusor contractions at a low bladder volume of 246 mL; an increased postvoid residual urine volume of 225 mL; and a slightly decreased urinary flow rate. Which of the following is not consistent with a normal age-associated change? A. PSA level of 3.3 ng/mL B. Decreased bladder capacity C. Involuntary detrusor contradictions D. Increased postvoid residual urine volume 13. Mrs. Smith, a 65-year-old woman presents to clinic for the first time and complains of urinary incontinence and dyspareunia. She went through menopause 10 years ago without any hormone replacement therapy and had a hysterectomy for a fibroid. Her mother had a hip fracture at 82 years of age. The patient’s most recent mammogram was 5 years ago and no known family history of breast cancer. She is not taking any medications. Her physical examination is unremarkable except for findings consistent with atrophic vaginitis. You decide to begin topical hormone replacement therapy. Which of the following evaluations would be necessary prior to initiating hormone replacement therapy? A. Mammogram B. Endometrial biopsy C. Bone mineral density measurement D. Papanicolaou smear 14. Mrs. L. Billings is a 77-year-old Caucasian female who has a history of breast cancer. She has been in remission for 6 years. As her primary care provider, you are seeing her for follow-up of her recent complaint of intermittent abdominal pain of a 3-month duration and some general malaise. Given the brief history above, what will you direct your assessment at during physical examination? A. Examination of her thyroid to rule out thyroid nodules that may contribute to her feeling fatigued. B. Auscultation of her abdomen for abnormal bowel sounds to rule out peritonitis. C. Thorough abdominal and gynecological exam to rule out masses and identify any tenderness. D. A rectal examination to rule out colon cancer as a secondary site for breast cancer. 15. A 78-year-old female comes to the office because she has pain when she urinates. She has been seen three times for this problem in the last 3 months. Each time she was told she had a UTI and was given antibiotics. She carefully followed the instructions but has had no relief of symptoms. Last UA: WBCs: 2-3/high-power field RBCs: 0-2/high-power field Epithelial cells: Few Nitrite: Negative Leuckocyte esterase: Negative Which of the following should be done next? A. Obtain a clean catch urine for UA and urine for C&S B. Perform a pelvic examination C. Reassure the patient that she has asymptomatic bacteriuria and does not need antibiotics D. Order a pelvic ultrasound Chapter 12. Musculoskeletal Disorders Multiple Choice Identify the choice that best completes the statement or answers the question. 1. Osteoarthritis of the cervical and lumbar spine causes pain that is related to all of the following except: A. Bone spur formation B. Pressure of the ligaments C. Reactive muscle spasm D. Crystal deposition 2. In differentiating osteoarthritis from chronic gout, pseudogout, or septic arthritis, the most valuable diagnostic study would be: A. Erythrocyte sedimentation rate (ESR) B. Synovial fluid analysis C. C-reactive protein analysis D. Complete blood cell count 3. Patients with osteoarthritis of the hip and knee often have a distinguishable gait described as: A. Ataxic B. Festinating C. Antalgic D. Steppage 4. Which of the following best describes the pain associated with osteoarthritis? A. Constant, burning, and throbbing with an acute onset? B. Dull and primarily affected by exposure to cold and barometric pressure C. Begins upon arising and after prolonged weight bearing and/or use of the joint D. Begins in the morning but decreases with activity 5. Joint effusions typically occur later in the course of OA, especially in the: A. Knee B. Elbow C. DIP joints D. Hips 6. You have ordered a CBC for your patient you suspect has polymyalgia rheumatica (PMR). Which two clinical findings are common in patients with PMR? A. Neutropenia and hypochromic, normocytic anemia B. Normochromic, normocytic anemia and thrombocytosis C. Microcytic, hypochromic anemia and reticulocytopenia D. Macrocytic, hyperchromic anemia and leukocytopenia 7. You suspect that your patient has polymyalgia rheumatica and now are concerned that the patient may also have Giant Cell Arteritis (GCA). Which of the following two symptoms are most indicative of GCA and PMR? A. Jaw pain and heart murmur B. Joint swelling and sudden loss of central vision bilaterally C. Hoarseness and the total inability to grasp small objects D. Scalp tenderness and aching in shoulder and pelvic girdle 8. Your 63-year-old Caucasian woman with polymyalgia rheumatica (PMR) will begin treatment with corticosteroids until the condition has resolved. You look over her records and it has been 2 years since her last physical examination and any laboratory or diagnostic tests as she relocated and had not yet identified a health-care provider. In prioritizing your management plan, your first orders should include: A. Recommending she increase her dietary intake of Calcium and Vitamin D B. Ordering once a year bisphosphonate and a proton pump inhibitor C. Participate in a fall prevention program D. Dual-energy x-ray (DEXA) scan and updating immunizations 9. Which of the following differential diagnosis for patients presenting with polymyalgia rheumatica (PMR) can be ruled out with a muscle biopsy? A. Parkinson’s disease B. Polymyositis C. Late onset rheumatoid arthritis D. Giant Cell Arteritis 10. In reviewing laboratory results for patients suspected with polymyalgia rheumatica (PMR), you realize that there is no definitive test to diagnosis PRM, rather clinical response to treatment. Results you would expect to see include: A. Elevated erythrocyte sedimentation rate (ESR) greater than 50mm per hour B. Elevated rheumatoid factor and anti-citrullinated protein antibodies (RF and ACPA) C. Decreased C-reactive protein level (CR-P) D. Elevated thyroid stimulating hormone (TSH) 11. Which of the following is the most appropriate laboratory test for monitoring gout therapy over the long-term? A. Erythrocyte sedimentation rate (ESR) B. Completer blood count (CBC) C. Serum urate level D. Serum albumin 12. In providing health teaching related to dietary restrictions, the nurse practitioner should advise a patient with gout to avoid which of the following dietary items: A. Green leafy vegetables B. Beer, sausage, fried seafood C. Sugar D. Gluten and bread items 13. The best method of verifying a diagnosis of gout in a joint is which of the following: A. Radiographic examination of the joint with two views B. Ultrasound C. Palpation D. Joint aspiration and polarized-light microscopy 14. The most appropriate first-line treatment for an acute gout flare is (assuming no kidney disease or elevated bleeding risk): A. Indomethacin 50 mg TID for 2 days; then 25 mg TID for 3 days B. Doxycycline 100 mg BID for 5 days C. Prednisolone 35 mg QD for 5 days D. Ice therapy 15. The nurse practitioner orders bilateral wrist X-rays on a 69-year-old gentleman complaining of pain in both wrists for the past 6 weeks not related to any known trauma. The nurse practitioner suspects elderly onset rheumatoid arthritis. The initial radiographic finding in a patient with elderly onset rheumatoid arthritis would be: A. Symmetric joint space narrowing B. Soft tissue swelling C. Subluxations of the joints D. Joint erosions 16. The nurse practitioner is examining the hands of a 55-year-old woman with rheumatoid arthritis and notes bilateral spindle shaped deformities on the middle interphalangeal joints. These are known as: A. Haygarth’s nodes B. Heberden’s nodes C. Bouchard’s nodes D. Benediction hands 17. A 72-year-old female patient has been diagnosed with gout. She also has a long history of chronic congestive heart failure. The most likely contributing factor to the development of gout in this older female is: A. Lead intoxication B. Illegal whiskey C. Binge-eating D. Thiazide diuretics 18. Which of the following statements about osteoarthritis is true? A. It affects primarily weight-bearing joints B. It is a systemic inflammatory illness C. The metacarpal phalangeal joints are commonly involved D. Prolonged morning stiffness is common 19. In considering the specificity of laboratory data, the most reliable diagnostic test listed below would be: A. Elevated erythrocyte sedimentation rate (ESR) to rule out inflammation B. CBC to rule out infection C. Antinuclear Antibody (ANA) test to rule out a collagen disease D. Synovial fluid analysis to differentiate between an infectious versus an inflammatory infusion 20. When examining the spine of an older adult you notice a curvature with a sharp angle. This is referred to as a: A. Gibbus B. Scoliosis C. Kyphosis D. Lordosis Chapter 13. Central and Peripheral Nervous System Disorders Chapter 13: Central and Peripheral Nervous System Disorders Multiple Choice Identify the choice that best completes the statement or answers the question. 1. An older adult patient is being evaluated following a stroke, which affected the anterior cerebral circulatory system. Which of the following signs/symptoms would be positive in the assessment of this patient? A. Bilateral vision disturbance and/or diplopia B. Dysarthria and speech difficulties C. Disorders in behavior and in cognition D. Motor and sensory problems on both sides of the body 2. When assessing a patient who complains of a tremor, the nurse practitioner must differentiate essential tremor from the tremor of Parkinson’s disease. Which of the following findings are consistent with essential tremor? B. Frontal lobe C. Temporal lobe D. Parietal lobe 7. An elderly patient has had a CVA in the anterior cerebral circulatory system (frontal lobe). What symptoms are most likely expressed? A. Neglect of body and difficulty organizing space B. Wernicke’s aphasia (difficulty understanding speech) C. Disorders of behavior and cognition D. Bilateral motor and sensory problem 8. The most common neurological cause of seizures in an older adult is: A. Alzheimer’s disease B. Multiple sclerosis C. Stroke D. Peripheral neuropathy 9. You are evaluating an elderly patient’s complaint of a new onset of “hand shaking” when he drinks from a cup. The patient is euthyroid and does not drink alcohol. Which of the following signs indicate that this tremor is most likely an essential tremor rather than a Parkinson’s tremor? A. It is characteristic of “pill rolling” and occurs at rest B. It is a moderate amplitude tremor that occurs only with movement/activity C. The tremor is also associated with slowed movements D. The tremor is accompanied by dyskinesia 10. Mr. Andrews experienced a brief onset of right-sided weakness, slurred speech, and confusion yesterday. The symptoms have resolved. What should the nurse practitioner do? A. Assure the patient that he will not experience the symptoms again B. Identify modifiable cardiovascular risk factors C. Do a thorough medication review and a CT scan D. Order a stat EEG and administer O2 by mask 11. An older male patient is experiencing acute onset of right-sided weakness, slurred speech, and confusion. What should the nurse practitioner do promptly? A. Administer an aspirin by mouth B. Evaluate for stroke and arrange transport to the hospital right away C. Do a thorough medication review and stat blood sugar D. Order an EKG and administer O2 by cannula immediately 12. An older adult female patient had a stroke. What symptoms are not usually expressed by patients who have had a vertebrobasilar stroke? A. Vertigo B. Ataxia C. Dysarthria D. Monocular blindness 13. When assessing an elderly client who reports a tremor, which assessment findings would be most reliable in identifying Parkinson’s disease? A. Any presence of tremor B. Symptoms of slowed movement, unstable angina, and tremor C. Resting tremor, slow unsteady gait, and cogwheel resistance D. Cogwheel rigidity, bradykinesia, and amnesia 14. A 78-year-old man with Parkinson’s disease is being cared for in a nursing home. The nurses observe that he coughs at mealtime; he has the ability to still feed himself with adaptive equipment. He has had no aspirations. Oral exam is normal, except during his swallowing assessment you note there is slight delayed elicitation. You assess that he has only mild dysphagia. You recommend which of the following as the next step in his care? A. Consult gastroenterologist for g-tube decision B. Consult speech pathologist for video fluoroscopy C. Provide a trial of foods of differing consistency D. Require that he is fed all his meals 15. Which of the following assessment findings is commonly noted in a client with advanced Parkinson’s disease? A. Macrographia B. Micrographia C. Ataxic gait D. Increased arm swing 16. A middle-aged patient has been diagnosed with Parkinson’s disease. What influences the nurse practitioner’s decision to begin pharmacological treatment for this patient? A. Intentional tremors B. Gait instability requiring use of a cane C. Symptoms interfering with functional ability D. Medications initiated at first sign of unilateral involvement 17. An older male presents with incontinence, has a wide-based and irregular gait, and progressive dementia. Which of the following presents with these cardinal signs? A. Parkinson’s disease B. Normal pressure hydrocephalus C. Basilar artery CVA D. Progressive supranuclear palsy 18. In review of a nursing home patient’s chart, the nurse practitioner discovers that the neurology consult identified that the client has a homonymous hemianopsia. Which of the following statements accurately describes these findings? A. Partial loss of visual acuity in the ipsilateral peripheral fields B. Diplopia in the eye contralateral to the cerebral lesion C. Loss of vision in either the right or left halves of the visual fields D. Nystagmus in both eyes with asymmetrical movement 19. The Mini-Cog is a short screening tool used to assess cognition. Which of the following statements pertaining to the test is a true statement? A. The patient will be asked to repeat five words immediately following the directions by the practitioner. B. The patient is asked to draw the hour and minute hands on a picture of an analog clock. C. A score of 0-2 is a positive screen for dementia. D. The patient is asked to recall five images from picture cards following the drawing of the clock hands. Chapter 14. Endocrine, Metabolic, and Nutritional Disorders Multiple Choice Identify the choice that best completes the statement or answers the question. 1. An elderly client presents with a new onset of feeling her heart race and fatigue. An EKG reveals atrial fibrillation with rate >110. The patient also has a new fine tremor of both hands. Which of the following would the nurse practitioner suspect? A. Hypothyroidism B. Hyperthyroidism C. Congestive heart failure D. Type 2 diabetes mellitus 2. A 62-year-old female complains of fatigue and lack of energy. Constipation has increased and the patient has gained ten pounds in the past 3 months. Depression is denied although the patient reports a lack of interest in usual hobbies. Vital signs are within normal limits and the patient’s skin is dry and cool. Which of the following must be included in the differential? A. Hyperthyroidism B. Hypothyroidism C. Hyperparathyroidism D. Grave’s disease 3. Mrs. Black, an 87-year-old patient, has been taking 100 mcg of Synthroid for 10 years. She comes to your office for a routine follow-up, feeling well. Her heart rate is 90. Your first response is to: A. Increase the Synthroid B. Order TSH C. Start a beta-blocker D. Order thyroid scan 4. Which patient is most likely to have osteoporosis? A. An 80-year-old underweight male who smokes and has been on steroids for psoriasis B. A 90-year-old female with no family history of osteoporosis who is on hormone replacement therapy C. A 68-year-old overweight female who drinks 1-2 drinks alcohol/day D. An 82-year-old female with a normal BMI takes calcium and performs weight- bearing exercise daily 5. When evaluating the expected outcome for a hypothyroid elderly patient placed on levothyroxine, the nurse practitioner will: A. Assess a weekly TSH B. Assess the TSH in 4-6 weeks C. Ask the patient if the symptoms have subsided and adjust dosage accordingly D. Decrease the dosage should a cardiac event occur 6. A postmenopausal woman with osteoporosis is taking a bisphosphonate daily by mouth. What action information statement would indicate the patient understood the nurse practitioner’s instructions regarding this medication? A. Takes medication at bedtime with a full glass of water B. Takes medication with a full glass of water when up in a.m. 30 minutes prior to other food and medications C. Takes medication when up in a.m. with a glass of orange juice to increase absorption D. Takes medication sitting up and with a meal to avoid gastrointestinal distress 7. The primary reason levothyroxine sodium is initiated at a low dose in an elderly patient with hypothyroidism is to prevent which of the following untoward effects? A. Angina and arrhythmia B. Nausea and diarrhea C. Confusion and delirium D. Osteoporosis and muscle weakness 8. Six months ago an elderly patient was diagnosed with subclinical hypothyroidism. Today the patient returns and has a TSH of 11.0 and complains of fatigue. He has taken Synthroid 25 mcg daily as prescribed. What is the best course of action for the nurse practitioner? A. Assess further for a cause of fatigue B. Double the dose of Synthroid C. Discontinue the Synthroid D. Prescribe Liotrix (T3 & T4 combination) 9. A fluoroquinolone (Ciprofloxin) is prescribed for a male patient with a UTI. What should the nurse practitioner teach the patient regarding taking this medication? A. It must be taken on an empty stomach B. Its effectiveness is decreased by antacids, iron, or caffeine ingestion C. It may cause a metallic aftertaste D. Its effectiveness is not a concern and it can be taken with any medications 10. A patient has been prescribed metformin (Glucophage). One week later he returns with lowered blood sugars but complains of some loose stools during the week. How should the nurse practitioner respond? A. Discontinue the medication immediately B. Reassure the patient that this is an anticipated side effect C. Double the dosage of medication and have patient return in 1 week D. Order a chem. 7 to check for lactic acidosis 11. Which of the following signs of hyperthyroidism commonly manifest in younger populations, but is notably lacking in the elderly? A. Weight gain B. Constipation C. Bradycardia D. Exophthalmos 12. A 60-year-old obese male client has type 2 diabetes mellitus and a lipid panel of TC = 250, HDL = 32, LDL = 165. The nurse practitioner teaches the patient about his modifiable cardiac risk factors, which include: A. Advancing age, diabetes, hyperlipidemia, and male gender B. Diabetes, obesity, and hyperlipidemia C. Hyperlipidemia, smoking, and family history of heart disease D. Male with age > 45, diabetes mellitus, and hyperlipidemia 13. A diabetic patient presents with the complaint of right foot pain but denies any recent known injury. He states it has gotten progressively worse over the past few months. On exam, vibratory sense, as well as sensation tested with a monofilament, was abnormal. The patient’s foot is warm, edematous, and misshapen. The nurse practitioner suspects Charcot foot. What intervention is indicated? A. Warm soaks and return for follow-up in 1 week B. Referral to a pain management clinic C. Referral to an orthopedist D. Referral to a cardiologist for evaluation of peripheral vascular disease 14. What is a sign of insulin resistance that can present in African American patients? A. Acanthosis Nigricans B. Psoriasis Nigricans C. Seborrheic Nigricans D. Bullemic Nigricans 15. During a routine physical examination of a 62-year-old female patient, the nurse practitioner identifies xanthelasma around both his eyes. What is the significance of this finding? A. High potential for future blindness and requires immediate referral B. None, normal variant of aging process C. Abnormal lipid metabolism requiring medical management D. Hereditary variant that is of no consequence but requires watchful waiting 16. Mr. White is 62 years old and has chronic kidney disease that has been relatively stable. He also has a history of hyperlipidemia, osteoarthritis, and hypertension. He is compliant with his medications, and his BP has been well controlled on a calcium channel blocker. His last lipid panel showed: TC = 201, HDL = 40, TG = 180, LDL = 98. He currently takes Crestor 20 mg daily. In the office today, his BP is 188/90, and his urine dip now shows significant proteinuria. He denies any changes in his dietary habits or medication regimen. What would be the best medication change for Mr. White at this point? A. No change—have him return in 4 weeks for a re-check of his blood pressure and urine B. Increase the dose of the calcium channel blocker for his hypertension C. Change the calcium channel blocker to an ACE-I D. Increase the dose of his Crestor and have him return in 3 months for a re-check of his BP 17. You are working as a nurse practitioner in the Fast Track of the emergency room. A 76-year-old male presents with left upper quadrant abdominal pain. There can be many conditions that present as left upper quadrant pain, but which of the following is least likely to cause pain in the left upper quadrant? A. Splenic abscess B. Left pyelonephritis C. Splenic rupture D. Acute pancreatitis 18. Which is a “cardinal feature” of failure to thrive? A. Decline in toileting ability B. Gait disturbance C. Poor nutritional status D. Spiritual distress 19. Feeding gastrostomy tubes at end-of-life Alzheimer’s disease patients have been associated with: A. Prolongation of life B. Aspirational pneumonia C. Increase in skin infections D. Reversal of any clinical signs of failure to thrive 20. Which of the following nutritional indicators is not an indication of poor nutritional status in an older person? A. Albumin of < 3.5 g/dl B. Body mass index of 25 C. Total cholesterol < 150 mg/dl D. Loss of 10% body weight in 180 days Chapter 15. Hematologic and Immune System Disorders Multiple Choice Identify the choice that best completes the statement or answers the question. 1. Iron Deficiency Anemia (IDA) is classified as a microcytic, hypochromic anemia. This classification refers to which of the following laboratory data? A. Hemoglobin and hematocrit B. Mean Corpuscular Volume (MCV) and Mean Corpuscular Hemoglobin (MCH) C. Serum ferritin and serum iron D. Total iron binding capacity and transferrin saturation 2. Initial therapy for patients with IDA is: A. Blood transfusion B. Increasing dietary intake of green leafy vegetables C. Ferrous sulfate 325 mg orally bid-tid D. Parenteral administration of Venofer 3. Mrs. G, a 70-year-old patient, presents to you with a hemoglobin of 9.4 and a decreased MCV/MCV. You diagnose her with IDA and start her on ferrous sulfate 325 mg PO tid. The next step in your work-up would be: A. Send Mrs. G for a GI consult B. Obtain hemoglobin one week after oral therapy C. Schedule Mrs. G for an upper GI series D. Obtain stool for occult blood 4. Despite adherence to oral iron therapy of 2 weeks, your elderly patient with IDA demonstrates a drop in his hemoglobin from 8.4 to 7.4. He is complaining of fatigue, dyspnea, and heart racing. You would change his therapy by which of the following: A. Schedule patient for outpatient parenteral iron infusion B. Switch to a different form of oral therapy C. Schedule patient for outpatient blood transfusion D. Make no changes until evaluated by GI specialist 5. You are prescribing oral iron therapy for a patient with IDA. In teaching your patient about iron therapy, all of the following instructions should be reviewed except: A. Iron is best taken on an empty stomach one hour before meals B. Meat, fish, poultry, beans, and green leafy vegetables are good dietary sources of iron C. Adding vitamin D daily enhances absorption of iron D. Common GI side effects of iron therapy include constipation, nausea, and black stools 6. When interpreting laboratory data, you would expect to see the following in a patient with Anemia of Chronic Disease (ACD): A. Hemoglobin <12 g/dl, Mean Corpuscular Volume (MCV) decreased, Mean Corpuscular Hemoglobin (MCH) decreased B. Hemoglobin >12 g/dl, Mean Corpuscular Volume (MCV) increased, Mean Corpuscular Hemoglobin (MCH) increased C. Hemoglobin <12 g/dl, Mean Corpuscular Volume (MCV) normal, Mean Corpuscular Hemoglobin (MCH) normal D. Hemoglobin >12 g/dl, Mean Corpuscular Volume (MCV) decreased, Mean Corpuscular Hemoglobin (MCH) increased 7. The pathophysiological hallmark of ACD is: A. Depleted iron stores B. Impaired ability to use iron stores C. Chronic uncorrectable bleeding D. Reduced intestinal absorption of iron 8. The main focus of treatment of patients with ACD is: A. Replenishing iron stores B. Providing for adequate nutrition high in iron C. Management of the underlying disorder D. Administration of monthly vitamin B12 injections 9. In addition to the CBC with differential, which of the following laboratory tests is considered to be most useful in diagnosing ACD and IDA? A. Serum iron B. Total iron binding capacity C. Transferrin saturation D. Serum ferritin 10. Education of patient and family regarding ACD includes discussion of all of the following except: A. Early prevention of chronic conditions through healthy lifestyle measures B. Importance of adherence to the treatment plan for management of chronic conditions C. Explaining the ACD is mainly attributed to the aging process D. Importance of regular follow-up to monitor responses to treatments 11. Symptoms in the initial HIV infection include all of the following except: A. Sore throat B. Fever C. Weight loss D. Headache 12. The primary mode of HIV transmission in the U.S. is: A. Blood transfusion B. Men having sex with men C. Heterosexual contact D. Needle sharing 13. In the older adult, which group comprises the highest rate of HIV/AIDS? A. African American women B. African American men C. Hispanic women D. Hispanic men 14. Contributing factors in HIV development in the older adult include all of the following except: A. Older adults are less likely to be tested B. Healthcare providers do not routinely ask older adults about HIV risks C. Older adults are less likely to use condoms D. Older adults are at low risk to contract HIV 15. Immune system changes that render older adults more susceptible to contracting HIV include all of the following except: A. Hyper-responsive T cells B. Age-related thymic involution C. Depletion of naïve CD4 cells D. Decline in B cell function Chapter 16. Psychosocial Disorders Multiple Choice Identify the choice that best completes the statement or answers the question. 1. The prevalence of depression in nursing home residents is greater than adults living in the community. A. 1-2 times B. 2-3 times C. 3-4 times D. 5 times 2. The majority of depressed older adults remain untreated because of: A. Misdiagnosis B. Social stigma C. Environmental barriers D. All of the above 3. Symptoms of depression distinct to the elderly include: A. Flat affect B. Loss of pleasure in usual activities C. Appetite and weight disturbances D. Lack of emotions 4. The justification for ordering a CBC, TSH, and serum B12 for a patient you suspect may have clinical depression is: A. To determine the cause of sadness B. Because of overlapping symptoms with anemia, thyroid dysfunction, and nutritional deficiencies 11. In late stages of dementia, a phenomenon called “sun downing” occurs, in which cognitive disturbances tend to: A. Improve as the day goes on B. Become worse toward the evening C. Fluctuate during the course of the day D. Peak mid-day 12. Of the following, which one is the most useful clinical evaluation tool to assist in the diagnosis of dementia? A. Folstein’s Mini-Mental Status Exam (MMSE) B. St. Louis University Mental Status Exam (SLUMS) C. Montreal Cognitive Assessment (MoCA) D. Geriatric Depression Scale (GDS) 13. The cornerstone of pharmacotherapy in treating Alzheimer’s disease is: A. Cholinesterase inhibitors B. NMDA receptor antagonist C. Psychotropic medications D. Anxiolytics 14. The comorbid psychiatric problem with the highest frequency in dementia is: A. Anxiety B. Depression C. Agitation and aggression D. Psychosis 15. When treating depression associated with dementia, which of the following would be a poor choice and should not be prescribed? A. Fluoxetine B. Desipramine C. Amitriptyline D. Mirtazapine Chapter 17. Polypharmacy MULTIPLE CHOICE 1. The nurse plans to closely monitor an older adult client who is receiving multiple drugs for evidence of adequate excretion of the medications. Which two major organs affecting drug clearance from the body should be monitored when an older adult client is receiving multiple drugs? a. Kidneys and lungs b. Kidneys and pancreas c. Liver and pancreas d. Liver and kidneys ANS: D The older adult client may experience a decrease in hepatic enzyme production, hepatic blood flow, and total liver function. These decreases cause a reduction in drug metabolism. A decrease in renal blood flow and decrease in glomerular filtration rate of 40% to 50% can also occur. 2. The nurse is aware that increased may affect the absorption of medications. a. gastric motility b. alkaline gastric secretion c. intestinal emptying time d. peristalsis ANS: B An increase in alkaline gastric secretions may affect the absorption of medications, because gastric motility, gastric emptying time, and peristalsis are all decreased with increased alkaline gastric secretions. 3. The older adult client is prescribed a drug that is excreted through the urine. The nurse anticipates that which laboratory test will be prescribed to evaluate renal function before administration of the medication? a. Blood urea nitrogen b. Serum albumin c. Serum electrolytes d. White blood cell count ANS: A Drug dosages are adjusted according to the older adults weight, adipose tissue, laboratory results (e.g., serum protein, electrolytes, liver enzymes, blood urea nitrogen, creatinine), and current health problems. 4. The nurse recognizes that polypharmacy is more likely to occur in an older adult client because the older client: a. has difficulty maintaining a list of current medications. b. experiences changes in visual capability. c. experiences alterations in mental capacity. d. may obtain prescribed drugs from various providers. ANS: D Polypharmacy is the prescription and administration of many drugs during the same period. This is more likely with the older client, who may see various healthcare providers to manage the health concerns that may accompany aging. 5. The diuretic hydrochlorothiazide (HCTZ; HydroDIURIL) is usually prescribed for the older adult in a lower dose. The nurse recognizes that this is an attempt to prevent the older adult from experiencing a(n): a. increase in potassium loss. b. decrease in sodium loss. c. decrease in blood sugar levels. d. increase in red and white blood cell count. ANS: A 6. The nurse notes that Cimetidine has been prescribed for an older adult client. The nurses highest priority would be to call: a. the physician. b. the pharmacist. c. the nursing supervisor. d. Poison Control. ANS: A Cimetidine is not recommended for the older adult because of its side effects and multiple potential drug interactions. Famotidine, nizatidine, and ranitidine may be prescribed for the geriatric client instead of cimetidine. 7. An older adult client says that he has difficulty removing the cap from his medication bottle. He says, Thats why I dont always take my medication. The nurse should suggest that he: a. ask a neighbor to help him remove the bottle cap. b. ask the pharmacist to put the drug in a nonchildproof container. c. have a family member visit daily to prepare his medications. d. ask the healthcare provider not to prescribe drugs that come in childproof bottles. ANS: B Childproof caps may be difficult to remove for older adults with arthritis or decreased strength or motor function. 8. A client is demonstrating signs and symptoms of disorientation. Which question by the nurse would be appropriate, related to the clients medication use? a. Are you taking any new medications? b. Do you take medications with a full glass of water? c. Do your medications make you constipated? d. Have you been diagnosed with dementia? ANS: A A new medication may be associated with changes in sensorium or other behavioral characteristics. This should be evaluated. 9. The nurse is caring for an older adult client and a middle-aged adult client. The nurse anticipates that the medication doses for the older adult will be those for the middle-aged adult. a. the same as b. less than c. slightly more than d. twice as much as ANS: B Because of changes in metabolism and excretion, doses are generally decreased to avoid accumulation and toxicity. 10. An older adult client is ordered to receive digoxin (Lanoxin). The client has decreased renal function. Which condition could indicate digoxin toxicity? a. Elevated blood pressure b. Decreased blood pressure c. Heart rate greater than 100 beats per minute d. Heart rate less than 60 beats per minute ANS: D Digoxin slows and strengthens the heart. Toxicity may be manifested by bradycardia. 11. A client takes ibuprofen to deal with the pain of arthritis. The client complains of gastrointestinal symptoms. Which is an appropriate strategy for the nurse to teach the client? a. Take the medication on an empty stomach. b. Do not take the medication if it has side effects. c. Try to take an antacid 1 hour before the dose. d. You must take the medication anyway. ANS: C An antacid may decrease the gastrointestinal side effects of the NSAID. It may also impair absorption, so it must be evaluated for effectiveness. 12. A nurse is administering medication to an older adult client. The nurse is aware that the clients half-life is longer than that of middle-aged adults. What nursing intervention is indicated by this premise? a. Provide higher doses to achieve therapeutic effects. b. Assess the client for adverse reactions. c. Monitor the client for signs of accumulation. d. Provide education to the client on drug reactions. ANS: C Because older adults may have less efficient medication elimination, monitoring for accumulation is an important component of nursing care. 13. The nurse is caring for an older adult client who complains that she has to take multiple medications at various times throughout the day. What is the most appropriate nursing intervention for the nurse to implement with this client? a. Encourage family members or friends to monitor drug regimen. b. Explain the purpose, drug action, and importance of medication. c. Develop a chart indicating times to take drugs. d. Provide time for questions; reinforce with written information. ANS: C The nurse should help the client develop a chart indicating times to take drugs. Space should be provided to mark each drug taken. An organizer or calendar can be used to mark days and weeks. 14. The physician is recommending a benzodiazepine medication for an older adult client who has previously been treated with Valium. The nurse recognizes that which medication would be most appropriate for this client? a. Librium b. Dalmane c. Ativan d. Equanil A short-acting benzodiazepine, such as Ativan, would be preferable to any of the other drugs to avoid prolonged sedation and addiction. 15. Which older adult client is most likely to experience digoxin toxicity? a. 72-year-old man who is also experiencing hyperkalemia b. 74-year-old woman who is also experiencing hypokalemia c. 66-year-old woman who is also experiencing hypercalcemia d. 62-year-old woman who is also experiencing hypocalcemia Chapter 18. Chronic Illness and the Advanced Practice Registered Nurse (APRN) Multiple Choice Identify the choice that best completes the statement or answers the question. 1. All of the following statements are true about chronic disease/illness except: A. Chronic disease is defined as a condition that requires both modification by the patient along with interaction with the provider B. The meaning of chronic illness includes the experience of the patient and family and provider C. Chronic illness is reversible if diagnosed and treated early D. Chronic conditions are those that last a year or longer 2. The highest number of people with multiple chronic conditions are those that are: A. Over age 85 B. Diagnosed with type 2 diabetes C. African American women over age 65 D. Diagnosed with COPD 3. Long durations of a chronic illness that are most commonly associated in both women and men are: A. Heart disease and obesity B. Diabetes and COPD C. Arthritis and dementia D. Dyslipidemia and eye disease 4. The percentage of non-institutionalized elderly over age 75 with limitations in activity due to a chronic condition is: A. 25% B. Nearly 50% C. Over 75% D. Less than 15% 5. Which of the following statements are true? A. Most Medicare participants have a single care provider B. Chronic kidney disease patients have better outcomes due to the dialysis benefit C. Nearly half of Medicare participants have five or more chronic diseases D. Nearly half of Medicare participants have three or more chronic diseases 6. Which statement about chronic mental disorders is false? A. The numbers are projected to increase in the next decade B. Individuals with mental health problems actually have a decreased risk of medical problems C. The number of older adults with addiction problems is projected to increase in the next decade D. Nearly half of individuals with dementia are not diagnosed prior to hospitalization 7. All of the following criteria define frailty except: A. Unintentional weight loss >10 pounds in the past year B. Slow walking speed C. Stooped posture D. Self-reported exhaustion 8. All of the following are health consequences of obesity except: A. Mortality of aging B. Hypertension C. Coronary artery disease D. Sleep apnea 9. Evidence-based clinical practice guidelines may have limited applicability in the older population because: A. They often are disease specific B. They often have complex comorbid conditions C. Both A and B D. None of the above evidence-based clinical practice guidelines are always the standard 10. Inter-professional care is distinguished from other models of team-based care in that: A. Independent providers share information B. Expert advice is shared from one provider to another C. Different disciplines work under one leader D. Shared leadership and accountability is a primary focus 11. All of the following statements about the Chronic Care Model are true except: A. It was developed with Robert Wood Johnson Foundation funding B. It focuses exclusively on a self-management model C. It is an evidence-based policy response designed to meet the needs of the chronically ill D. It directs quality improvement and system change for patients with chronic disease 12. The legislation signed in March 2010 with provisions for addressing chronic disease was: A. Accountable Care Organization Act B. Independence at Home Act C. Patient Protection and Affordable Care Act D. Patient Centered Medical Home Act 13. Reported outcome measures for the Patient Centered Medical Home include all of the following except: A. Emergency room use B. Hospitalizations C. Patient/family satisfaction D. Number of primary care visits 14. Transitional care is best described as: A. A set of actions to ensure the coordination and continuity of care across sites/settings B. The care received in transit from one site of care to another C. A discharge-planning model D. Long-term acute care that provides the transition from hospital to long-term care or home Chapter 19. Palliative Care and End-of-Life Care Multiple Choice Identify the choice that best completes the statement or answers the question. 1. Hospice care differs from palliative care in that: A. It is not covered by insurance B. Supports patients and families through both the dying and the bereavement process C. It cannot be provided in the nursing home D. The majority of those admitted to hospice die within 7 days 2. Evidence reflects the primary obstacle to implementing palliative care in the long-term care setting include all of the following except: A. Inadequate communication between decision makers B. Failure to recognize futile treatments C. Lack of advance directives D. Lack of sufficient staff 3. The most prevalent symptom in end-of-life care includes all of the following except: A. Lower extremity weakness B. Pain C. Dyspnea D. Delirium 4. Pain at the end of life is most often due to all of the following except: A. Musculoskeletal disorders B. Headache C. Cancer pain D. Neuropathic pain 5. The strongest level of evidence reflects the absolute contraindication for NSAID use exists with: A. Chronic kidney disease B. Peptic ulcer disease C. Heart disease D. Liver disease 6. The drug specific for severe opioid-induced constipation is: A. Lactulose B. Ducolax C. Mineral oil D. Methylnaltrexone 7. Pharmacological interventions for dyspnea include all of the following except: A. Antitussives B. Inhaled anesthetics C. Sedatives D. Anxiolytics 8. Delirium is typically characterized by all of the following except: A. Inattention B. Hyperactive level of psychomotor activity C. Disorganized thinking D. Altered level of consciousness 9. The first step in treating delirium is to: A. Give low-dose Haldol if the patient has psychosis B. Identify the cause C. Provide reality orientation D. Use side rails and/or wrist restraints to keep the patient safe 10. The majority of patients enrolled in hospice care die: A. In a nursing home B. In the hospital C. In an inpatient hospice facility D. At home 11. A drug that can be used to treat two very common symptoms in a dying patient (pain and dyspnea) is: A. Morphine B. Methadone C. Gabapentin D. Lorazepam 12. The tasks of grieving include all of the following except: A. Acknowledge the reality of death B. Work through the pain of grief C. Begin to disengage D. Restructure relationships 13. The best description of complicated grief is: A. Chronic, delayed, exaggerated, masked, or disenfranchised B. Experience of shock during notification of the death C. When grief is actually experienced before the death of a loved one D. A physical illness develops soon after death of a loved one 14. All of the following statements are true about interventions in working with the bereaved except: A. Allow the active expression of grief B. There is strong evidence behind recommended interventions C. Staff attending memorial services support the family D. Provide both emotional and spiritual support 15. The highest level of evidence to support interventions at the end of life is with: A. Opioids for pain control B. Hyocsyamine for respiratory secretions C. Stimulant laxatives for treating constipation D. Opioids for dyspnea 16. The highest level of evidence with the use of adjuvant analgesics is with: A. Tricyclic antidepressants avoided due to high adverse events B. Therapeutic trials before discontinuing drugs C. Patient with fibromyalgia are candidates for adjuvant analgesics D. Neuropathic pain patients are candidates for adjuvant analgesia [Show More]

Last updated: 1 month ago

Preview 1 out of 70 pages

Reviews( 2 )

user-profile-pic


by medman123 · 3 years ago

good

user-profile-pic


by Cguillory · 2 years ago

Great material!

Recommended For You

 Maternal Child Nursing Care> TEST BANK > Test Bank for Women's Health Care in Advanced Practice Nursing 2nd Edition by Ivy M. Alexander, Versie Johnson-Mallard, Elizabeth Kostas-Polston, Catherine Ingram Fogel & Nancy Fugate Woods - Complete, Elaborated and Latest Test Bank. ALL Chapters (1-46) Included and Updated (All)

preview
Test Bank for Women's Health Care in Advanced Practice Nursing 2nd Edition by Ivy M. Alexander, Versie Johnson-Mallard, Elizabeth Kostas-Polston, Catherine Ingram Fogel & Nancy Fugate Woods - Complete, Elaborated and Latest Test Bank. ALL Chapters (1-46) Included and Updated

Test Bank for Women's Health Care in Advanced Practice Nursing 2nd Edition by Ivy M. Alexander, Versie Johnson-Mallard, Elizabeth Kostas-Polston, Catherine Ingram Fogel & Nancy Fugate Woods - Complete...

By Prof.Exams , Uploaded: Apr 18, 2024

$32

 MATERANAL NURSING> TEST BANK > Test Bank for Women's Health Care in Advanced Practice Nursing 2nd Edition by Ivy M. Alexander, Versie Johnson-Mallard, Elizabeth Kostas-Polston, Catherine Ingram Fogel & Nancy Fugate Woods - Complete, Elaborated and Latest Test Bank. ALL Chapters (1-46) Included and Updated (All)

preview
Test Bank for Women's Health Care in Advanced Practice Nursing 2nd Edition by Ivy M. Alexander, Versie Johnson-Mallard, Elizabeth Kostas-Polston, Catherine Ingram Fogel & Nancy Fugate Woods - Complete, Elaborated and Latest Test Bank. ALL Chapters (1-46) Included and Updated

Test Bank for Women's Health Care in Advanced Practice Nursing 2nd Edition by Ivy M. Alexander, Versie Johnson-Mallard, Elizabeth Kostas-Polston, Catherine Ingram Fogel & Nancy Fugate Woods - Complete...

By ProfXams , Uploaded: Apr 18, 2024

$33

 *NURSING> TEST BANK > TEST BANK - Advanced Practice Nursing - 2020 - Primary Care 5ED BY Dunphy (All)

preview
TEST BANK - Advanced Practice Nursing - 2020 - Primary Care 5ED BY Dunphy

Primary Care: Art and Science of Advanced Practice Nursing - An Interprofessional Approach 5 th edition Dunphy Test Bank1. A nurse has conducted a literature review in an effort to identify the effect...

By ProfXams , Uploaded: Aug 07, 2021

$28

 *NURSING> TEST BANK > Primary Care Art and Science of Advanced Practice Nursing – An Interprofessional Approach 5ED Test Bank-2021 Lynne M. Dunphy , Jill E. Winland-Brown , Brian Oscar Porter , Debera J. Thomas (All)

preview
Primary Care Art and Science of Advanced Practice Nursing – An Interprofessional Approach 5ED Test Bank-2021 Lynne M. Dunphy , Jill E. Winland-Brown , Brian Oscar Porter , Debera J. Thomas

Primary Care Art and Science of Advanced Practice Nursing – An Inter-professional Approach 5ED Test Bank-2021 Lynne M. Dunphy , Jill E. Winland-Brown , Brian Oscar Porter , Debera J. Thomas A patient...

By ProfXams , Uploaded: Sep 08, 2021

$24.5

 *NURSING> TEST BANK > TEST BANK for Advanced Practice Nursing Essentials for Role Development 4th Edition by Joel (All)

preview
TEST BANK for Advanced Practice Nursing Essentials for Role Development 4th Edition by Joel

Advanced Practice Nursing Essentials for Role Development 4th Edition Joel Test Bank

By quizprof , Uploaded: Mar 03, 2021

$19

 *NURSING> TEST BANK > TEST BANK for Advanced Practice Nursing in the Care of Older Adults 2nd Edition by Laurie Malone, Lori Plank, Evelyn Duffy (Chapter 1-19) (All)

preview
TEST BANK for Advanced Practice Nursing in the Care of Older Adults 2nd Edition by Laurie Malone, Lori Plank, Evelyn Duffy (Chapter 1-19)

Advanced Practice Nursing in the Care of Older Adults/ Edition2 TESTBANK Chapter 1. Changes with Aging Multiple Choice Identify the choice that best completes the statement or answers the que...

By Expert#1 , Uploaded: May 26, 2020

$19

 *NURSING> TEST BANK > TEST BANK for Primary Care Art and Science of Advanced Practice Nursing - An Interprofessional Approach 5th Edition Lynne M. Dunphy (All)

preview
TEST BANK for Primary Care Art and Science of Advanced Practice Nursing - An Interprofessional Approach 5th Edition Lynne M. Dunphy

Primary Care: Art and Science of Advanced Practice Nursing - An Interprofessional Approach 5th edition Dunphy Test Bank|Test Bank Primary Care Art and Science of Advanced Practice Nursing - An Interpr...

By Expert1 , Uploaded: Sep 20, 2020

$22

 *NURSING> TEST BANK > Test Bank for Advanced Practice Nursing Essentials for Role Development 4th Edition by A. Joel All Chapters (All)

preview
Test Bank for Advanced Practice Nursing Essentials for Role Development 4th Edition by A. Joel All Chapters

Test Bank for Advanced Practice Nursing Essentials for Role Development 4th Edition by A. Joel All ChaptersTest Bank for Advanced Practice Nursing Essentials for Role Development 4th Edition by A. J...

By Gradexam , Uploaded: Mar 11, 2024

$20

 *NURSING> TEST BANK > Test Bank for Hamric and Hanson’s Advanced Practice Nursing An Integrative Approach 6th Edition Tracy O’Grady All Chapters (All)

preview
Test Bank for Hamric and Hanson’s Advanced Practice Nursing An Integrative Approach 6th Edition Tracy O’Grady All Chapters

Test Bank for Hamric and Hanson’s Advanced Practice Nursing An Integrative Approach 6th Edition Tracy O’Grady All Chapters Test Bank for Hamric and Hanson’s Advanced Practice Nursing An Integrative Ap...

By Gradexam , Uploaded: Mar 11, 2024

$20

 *NURSING> TEST BANK > Test Bank for Primary Care Art and Science of Advanced Practice Nursing An Interprofessional Approach 5th Edition All Chapters (All)

preview
Test Bank for Primary Care Art and Science of Advanced Practice Nursing An Interprofessional Approach 5th Edition All Chapters

Test Bank for Primary Care Art and Science of Advanced Practice Nursing An Interprofessional Approach 5th Edition All Chapters Test Bank for Primary Care Art and Science of Advanced Practice Nursing A...

By Gradexam , Uploaded: Mar 13, 2024

$20

$24.00

Add to cart

Instant download

Can't find what you want? Try our AI powered Search

OR

GET ASSIGNMENT HELP
1264
14

Document information


Connected school, study & course



About the document


Uploaded On

Sep 03, 2020

Number of pages

70

Written in

Seller


seller-icon
Expert#1

Member since 4 years

411 Documents Sold


Additional information

This document has been written for:

Uploaded

Sep 03, 2020

Downloads

 14

Views

 1264

THE BEST STUDY GUIDES

Avoid resits and achieve higher grades with the best study guides, textbook notes, and class notes written by your fellow students

custom preview

Avoid examination resits

Your fellow students know the appropriate material to use to deliver high quality content. With this great service and assistance from fellow students, you can become well prepared and avoid having to resits exams.

custom preview

Get the best grades

Your fellow student knows the best materials to research on and use. This guarantee you the best grades in your examination. Your fellow students use high quality materials, textbooks and notes to ensure high quality

custom preview

Earn from your notes

Get paid by selling your notes and study materials to other students. Earn alot of cash and help other students in study by providing them with appropriate and high quality study materials.


$24.00

WHAT STUDENTS SAY ABOUT US


What is Browsegrades

In Browsegrades, a student can earn by offering help to other student. Students can help other students with materials by upploading their notes and earn money.

We are here to help

We're available through e-mail, Twitter, Facebook, and live chat.
 FAQ
 Questions? Leave a message!

Follow us on
 Twitter

Copyright © Browsegrades · High quality services·